Skip to content

Commit

Permalink
LINMA2380: Update exercise solutions for 2020--2021
Browse files Browse the repository at this point in the history
  • Loading branch information
Peiffap committed Feb 2, 2021
1 parent 8ffb2e3 commit 7edd2bc
Show file tree
Hide file tree
Showing 9 changed files with 1,304 additions and 927 deletions.
607 changes: 396 additions & 211 deletions src/q7/matrix-INMA2380/exercises/ch1.tex

Large diffs are not rendered by default.

464 changes: 140 additions & 324 deletions src/q7/matrix-INMA2380/exercises/ch2.tex

Large diffs are not rendered by default.

364 changes: 203 additions & 161 deletions src/q7/matrix-INMA2380/exercises/ch3.tex
Original file line number Diff line number Diff line change
@@ -1,187 +1,229 @@
\section{Eigenvalues, eigenvectors and similitude}
\exo{1}
\begin{solution}
If $A \Ima(x) \subseteq \Ima(x)$, since $\Ima(x)$ and $A\Ima(x)$
are vector subspaces and $\Ima(x)$ is of dimension 1,
that means that either $A\Ima(x) = \Ima(x)$ or $A\Ima(x) = \{0\}$.
In the first case, it is an eigenvector of a nonzero eigenvalue.
In the second case, it is an eigenvector of 0.
\end{solution}
\section{Unitary transformations and the Singular Value Decomposition}

\exo{2}
\begin{solution}
Let's first show that if $A$ is normal,
it is still normal after an unitary transformation.

\begin{align*}
(U^TAU)^T(U^TAU)
& = U^TA^TAU\\
& = U^TAA^TU\\
& = U^TAUU^TA^TU\\
& = U^TAU(U^TAU)^T.
\end{align*}
That means that the Schur form of $A$ is normal too.
Assuming that all the diagonal elements of \(\Lambda\) are distinct, show that the matrix \(U_{up}\) is necessarily diagonal and consists only of phases:
\[
U_{up} = \diag\{e^{i\psi_1}, \dots, e^{i \psi_n}\}.
\]

We can now use the same argument as in theorem~3.3 since $A_s^* = A_s^T$
and apply it for all $n_1+n_2 = n$ such that $n_1$ is at the end of a block.
\begin{solution}
Let $U_{up} = [u_{ij}]_{i,j=1}^{n}$.
We must have
\[ U_{up}\Lambda = \Lambda U_{up}. \]
At the position $i,j$, we have
\[ u_{ij} \lambda_j = \lambda_i u_{ij} \]
or
\[ u_{ij} (\lambda_j - \lambda_i) = 0. \]
If $i \neq j$, this means that $u_{ij} = 0$ since $\lambda_j - \lambda_i \neq 0$.

$U_{up}$ must therefore be diagonal.
However it also needs to be hermitian for $UU_{up}$ to be Hermitian.
Indeed, we need to have
\begin{align*}
UU_{up} U_{up}^*U^* & = I\\
U_{up} U_{up}^* & = U^*U\\
U_{up} U_{up}^* & = I.
\end{align*}
This means that we need to have $1 = u_{ii}\overline{u_{ii}} = \abs{u_{ii}}^2$
so $u_{ii}$ is on the unit circle and is a phase.
\end{solution}

\exo{2}
Show that if \(X\) satisfies
\begin{itemize}
\item (1), then \(AX\) is a projection;
\item (1) and (3), then \(AX\) is an orthogonal projection;
\item (2), then \(XA\) is a projection;
\item (2) and (4), then \(XA\) is an orthogonal projection.
\end{itemize}

\begin{solution}
Let's first show that if $A$ is anti-symmetric,
it is still anti-symmetric after an unitary transformation.
\begin{itemize}
\item
\begin{align*}
AXAX
& = (AXA)X\\
& = AX.
\end{align*}
\item
$AX$ is a projection since (1) is satisfied.

From Theorem~3.6, $\Ker(AX) = \Ima((AX)^*)^\perp$.
Therefore, we can just show that $\Ima((AX)^*) = \Ima(AX)$ which is obvious
since (3) is satisfied.
\item
\begin{align*}
XAXA
& = (XAX)A\\
& = XA.
\end{align*}
\item
$XA$ is a projection since (2) is satisfied.

From Theorem~3.6, $\Ker(XA) = \Ima((XA)^*)^\perp$.
Therefore we can just show that $\Ima((XA)^*) = \Ima(XA)$ which is obvious
since (4) is satisfied.
\end{itemize}
\end{solution}

\begin{align*}
(U^TAU)^T
& = U^TA^TU\\
& = U^T(-A)U\\
& = -U^TAU.
\end{align*}
That means that the Schur form of $A$ is anti-symmetric too.
Since it is anti-symmetric, $A^TA = -A^2 = AA^T$ so it is also normal.
\exo{3}
How can we define the notion of canonical angles between two spaces of different dimension?

By the exercise~3.2, we know that the Schur form is block diagonal.
But since $A_s = -A_s^T$,
we must have $\alpha_j = -\alpha_j$ which means that $\alpha_j = 0$
(note that it imposes no restriction on $\beta_j$).
Same for $A_{ii}$.
\end{solution}
% i feel like there's some fuckery going on with matrix dimensions here :(

\exo{1}
\begin{solution}
We simply use the corollary since we remove the last line and column.

If the $\beta_i$ are nonzero, we can see that it is strict since
if $p_i(\lambda) = p_{i-1}(\lambda) = 0$, we have
\[ 0 = -\beta_i^2 p_{i-2}(\lambda) \]
so $p_{i-2}(\lambda) = 0$.
By induction, we have $p_0(\lambda) = 0$ which is absurd.
Let $\mathcal{S}_1$ be of dimension $m$ and $\mathcal{S}_2$ be of dimension $n < m$.
We can have $n$ canonical angles using
\begin{align*}
S_1^*S_2
& = U_1
\begin{pmatrix}
\Sigma\\0
\end{pmatrix}
U_2^*\\
& =
\begin{pmatrix}
U_{1,1} & U_{1,2}
\end{pmatrix}
\begin{pmatrix}
\Sigma\\0
\end{pmatrix}
U_2\\
& = U_{1,1} \Sigma U_2,
\end{align*}
with $\Sigma = \cos(\Theta)$, where $\theta_i$ are the angles between the $n$ vectors of $S_1U_{1,1}$
and the $n$ vectors of $S_2U_2$.
\end{solution}

\exo{2}
\exo{4}
Show that for every positive semidefinite matrix \(H\) and every unitary matrix \(Q\), we have
\[
\trace(HQ) \leq \trace(H).
\]

\begin{solution}
See syllabus.
You should obtain
\[ \sigma_k^2 - \|m_{i:}\|_2^2 \leq \hat{\sigma}_k^2 \leq \sigma_k^2 + \|m_{i:}\|_2^2 \]
and not
\[ \sigma_k^2 - \|m_{i:}\|_2^2 \leq \hat{\sigma}_k^2 \leq \sigma_k^2 \]
like written in the syllabus.
\emph{Hint:}
Use the property $\trace(AB) = \trace(BA)$.

Since $H$ is Hermitian, there is a unitary $U$ and diagonal $\Lambda$ such that $H = U \Lambda U^*$.
Let's first analyse the RHS
\begin{align*}
\trace(H) & = \trace(U \Lambda U^*)\\
& = \trace(\Lambda U^*U)\\
& = \trace(\Lambda)\\
& = \sum_{i=1}^n \lambda_i.
\end{align*}
For the LHS, we first need to analyse $Q$.
It is unitary, so its eigenvalues are on the unit circle therefore $\abs{x^* Q x} \leq 1$ for all $x$ such that $\norm{x}=1$.

Using the triangle inequality and the fact that the $\lambda_i$ are positive
\begin{align*}
\trace(HQ)
& = \trace(U \Lambda U^* Q)\\
& = \trace(\Lambda U^* Q U)\\
& = \sum_{i=1}^n \lambda_i u_{:i}^* Q u_{:i}\\
& \leq \abs{\sum_{i=1}^n \lambda_i u_{:i}^* Q u_{:i}}\\
& \leq \sum_{i=1}^n \abs{\lambda_i} \, \abs{u_{:i}^* Q u_{:i}}\\
& = \sum_{i=1}^n \lambda_i \abs{u_{:i}^* Q u_{:i}}\\
& \leq \sum_{i=1}^n \lambda_i.
\end{align*}
\end{solution}

\exo{3}
\begin{solution}
See syllabus.
\end{solution}
How could you extend the polar decomposition to the case \(m \neq n\)?

\exo{2}
\begin{solution}
See syllabus.

\begin{align*}
\sum_{i=0}^\infty \frac{(A+B)^i}{i!}
& = \sum_{i=0}^\infty \frac{\sum_{k=0}^i \frac{i!}{k!(i-k)!} A^{i-k}B^k}{i!}\\
& = \sum_{i=0}^\infty \sum_{k=0}^i \frac{A^{i-k}}{(i-k)!} \frac{B^k}{k!}\\
& =
\left(\sum_{i=0}^\infty \frac{A^i}{i!}\right)
\left(\sum_{i=0}^\infty \frac{B^i}{i!}\right).
\end{align*}
because each term $A^aB^b$ is present with the term $\frac{1}{a!b!}$.
If $A$ is $m \times n$, then $H$ is $m \times r$ and $Q$ is $r \times n$ for some $r$.
We want $H$ to be positive semidefinite so we need $H$ to be square for it to be defined.
That means that $Q$ is $m \times n$.

The problem with the proof when $m \neq n$ is that we cannot say
$U \Sigma V^* = U \Sigma U^* U V^*$ since the product is not defined because $U$ is $m \times m$
and $V$ is $n \times n$.
\begin{itemize}
\item If $m < n$, we have
\begin{align*}
A
& = U
\begin{pmatrix}
\Sigma & 0
\end{pmatrix}
\begin{pmatrix}
V_1 & V_2
\end{pmatrix}^*\\
& = U \Sigma V_1^*\\
& = (U \Sigma U^*) (U V_1^*)
\end{align*}
with $V_1^*V_1 = I$ (but $V_1V_1^* \neq I$, since $I$ has full rank but $V_1$ does not have full row rank)
since
\begin{align*}
I
& =
\begin{pmatrix}
V_1 & V_2
\end{pmatrix}^*
\begin{pmatrix}
V_1 & V_2
\end{pmatrix}\\
& =
\begin{pmatrix}
V_1^*V_1 & V_1^*V_2\\
V_2^*V_1 & V_2^*V_2
\end{pmatrix}.
\end{align*}

Therefore $QQ^* = (U V_1^*) (U V_1^*)^* = I$ but $Q^*Q = (U V_1^*)^* (U V_1^*) = V_1V_1^* \neq I$.
\item If $m > n$, we have
\begin{align*}
A
& =
\begin{pmatrix}
U_1 & U_2
\end{pmatrix}
\begin{pmatrix}
\Sigma \\ 0
\end{pmatrix}
V^*\\
& = U_1 \Sigma V^*\\
& = (U_1 \Sigma U_1^*) (U_1 V^*)
\end{align*}
with $U_1^*U_1 = I$ but $U_1U_1^* \neq I$ like for the previous point.

This time it's $Q^*Q = I$ and $QQ^* \neq I$.
\end{itemize}
\end{solution}

\exo{1}
\begin{solution}
We have

\begin{align*}
\begin{pmatrix}
\lambda_0 & 1 & & \\
& \ddots & \ddots & \\
& & \ddots & 1\\
& & & \lambda_0\\
\end{pmatrix}
& =
\lambda_0 I + N
\end{align*}
for
\[
N =
\begin{pmatrix}
& 1 & & \\
& & \ddots & \\
& & & 1\\
& & & \\
\end{pmatrix}.
\]
The rest is a simple consequence of the exercise~3.7.

It is important to note for the next page that $N^n = 0$.
This is indeed a consequence of the fact that $N = J - \lambda_0I$
where $J$ is a Jordan block of $\lambda_0$ of size $n$.
$J$ is therefore a matrix with only one eigenvalue $\lambda_0$
of algebraic multiplicity $n$ but geometric multiplicity $1$.
Hence the whole set $\mathbb{C}^n$ is an invariant subspace of $N$
which means that $(J - \lambda_0I)^n = 0$.

We can see for example for $n = 4$ that

\begin{align*}
N & =
\begin{pmatrix}
0 & 1 & 0 & 0\\
0 & 0 & 1 & 0\\
0 & 0 & 0 & 1\\
0 & 0 & 0 & 0
\end{pmatrix}\\
N^2 & =
\begin{pmatrix}
0 & 0 & 1 & 0\\
0 & 0 & 0 & 1\\
0 & 0 & 0 & 0\\
0 & 0 & 0 & 0
\end{pmatrix}\\
N^3 & =
\begin{pmatrix}
0 & 0 & 0 & 1\\
0 & 0 & 0 & 0\\
0 & 0 & 0 & 0\\
0 & 0 & 0 & 0
\end{pmatrix}\\
N^4 & =
\begin{pmatrix}
0 & 0 & 0 & 0\\
0 & 0 & 0 & 0\\
0 & 0 & 0 & 0\\
0 & 0 & 0 & 0
\end{pmatrix}
\end{align*}
We can see that $e_1$ is an eigenvector of $\lambda_0$ but
$e_2$, $e_3$, $e_4$ are not.
\begin{itemize}
\item $Je_2 = \lambda_0 e_2 + e_1$
so $(J - \lambda_0 I)e_2 = e_1$.
\item $(J - \lambda_0 I)e_3 = e_2$ and $(J - \lambda_0 I)^2e_3 = e_1$.
\item $(J - \lambda_0 I)e_4 = e_3$, $(J - \lambda_0 I)^2e_3 = e_2$
and $(J - \lambda_0 I)^3e_4 = e_1$.
\end{itemize}

Note that $J^4 \mathbb{C}^n = \mathbb{C}^n$ if $\lambda_0 \neq 0$.
It is not to be mistaken from $(J - \lambda_0 I)^4 \mathbb{C}^n = \{0\}$.
\end{solution}
\exo{3}
Show that the polar decomposition of \(B^\top A\) leads to an optimal rotation \(Q\) that minimizes \(\norm{AQ^\top - B}_F^2\).

\exo{1}
\begin{solution}
\begin{align*}
\fdif{}{t}e^{At}
& = \fdif{I}{t} + \sum_{i=1}^\infty \frac{A^i}{i!} \fdif{t^i}{t}\\
& = \sum_{i=1}^\infty \frac{iA^it^{i-1}}{i!}\\
& = A\sum_{i=1}^\infty \frac{(At)^{i-1}}{(i-1)!}\\
& = A \sum_{i=0}^\infty \frac{(At)^i}{i!} = A e^{At}\\
& = \sum_{i=0}^\infty \frac{A^{i+1} t^i}{i!}\\
& = \sum_{i=0}^\infty \frac{(At)^i}{i!} A = e^{At} A.
\end{align*}
\emph{Hint} Use the property $\norm{A}_F = \trace(A^\top A) = \trace(AA^\top)$ and the property $\trace(AB) = \trace(BA)$.

We have (also using $\trace(A^\top) = \trace(A)$).

\begin{align*}
\norm{AQ^\top - B}_F^2
& = \trace((AQ^\top - B)(AQ^\top - B)^\top)\\
& = \trace((AQ^\top - B)(QA^\top - B^\top))\\
& = \trace(AQ^\top QA^\top) + \trace(BB^\top) - \trace(AQ^\top B^\top ) - \trace(BQA^\top)\\
& = \trace(AA^\top) + \trace(BB^\top) - \trace(B^\top AQ^\top) - \trace(QA^\top B)\\
& = \norm{A}_F^2 + \norm{B}_F^2 - \trace(B^\top AQ^\top) - \trace(B^\top AQ^\top)\\
& = \norm{A}_F^2 + \norm{B}_F^2 - 2\trace(B^\top AQ^\top)\\
& = \norm{A}_F^2 + \norm{B}_F^2 - 2\trace(\tilde{H}\tilde{Q} Q^\top)\\
& \geq \norm{A}_F^2 + \norm{B}_F^2 - 2\trace(\tilde{H})
\end{align*}
using Exercise~3.4 since $\tilde{Q}Q^\top$ is unitary and $\tilde{H}$ is Hermitian positive semidefinite.
Taking $Q = \tilde{Q}$, we have equality. It is therefore the optimal rotation.
\end{solution}

\exo{1}
\exo{3}
Construct a matrix \(B\), with \(\mathop{\mathrm{rank}}(B) \leq s\), that is different from (3.15) and reaches the same bound on \(\norm{A - B}\).

\begin{solution}
See syllabus.
\end{solution}
For $s_i \in [\sigma_i - \sigma_{s+1}(A), \sigma_i + \sigma_{s+1}(A)]$ for $i = 1, \ldots, s$,
and
\[ B = \sum_{i=1}^s \mathbf{u}_i s_i \mathbf{v}_i^\top, \]
we have
\[ A - B = \sum_{i=1}^s \mathbf{u}_i(\sigma_i - s_i)\mathbf{v}_i^\top + \sum_{i=s+1}^r \mathbf{u}_i\sigma_i \mathbf{v}_i^\top. \]
which also has its maximum singular value equal to $\sigma_{s+1}(A)$ by definition of the $s_i$.
\end{solution}
Loading

0 comments on commit 7edd2bc

Please sign in to comment.